2010-11-17

請問有關代數和偏序的問題

1.P9-120 範例3 解答第3行k x k = k,這是因為k為zero, 0 * 0 = 0的關係嗎?

2.p10-26 範例10 (b) 若個數n=3,是不是有{{1},{12},{123}}.......{{1},{13},{123}}.......{{2},{12},{123}}.......{{2},{23},{123}}.......{{3},{13},{123}}.......{{3},{23},{123}} 這六種呢?有點不大懂。還有(d)和(e)可否麻煩解釋一下,挺複雜的

2 則留言:

線代離散助教(wynne) 提到...

1. yes

2. (b) 和你想的一樣

(d) 題目寫 "Let A be an antichain of S and ..." 裡面的 A 應改成那個長得比較炫的 A, 我在這裡以符號 A' 來表示炫A

這邊大致的意思是, 假設認取一個 A' 中的 element A, 其 size 為 k, 把它利用 (c) 裡的方法來變成一個 S 的 maximal chain, 也就是先拿那 k 個 antichain 中的 elements 建出一個互有保含於的關係, 有 k! 種可能, 然後再用剩下的 n-k 個 S-A 中的 elements 再擴充成一個長度為 n 的 maximal chain, 這樣這邊總共就有 k!(n-k)! 個 antichain; 把上面的步驟拿來對每一個 A' 中的 elements 都做一次, 即可造出 ∑(α_k)k!(n-k)! 這麼多條 maximal chains, 而因為 maximal chains 的總數由 (b) 可知不會超過 n!, 那這樣就得證了

(e) 我想你應該不會有太大的問題,
這裡只是把 (d) 的式子拿來推導而已

Sean 提到...

感謝助教的回答~~我了解了~~
原來是只取一個元素A,而這
種元素有αk個